K
Khách

Hãy nhập câu hỏi của bạn vào đây, nếu là tài khoản VIP, bạn sẽ được ưu tiên trả lời.

9 tháng 10 2017

Ta có \(a^2b^2+b^2c^2+c^2a^2\geq a^2b^2c^2\Leftrightarrow \frac{1}{a^2}+\frac{1}{b^2}+\frac{1}{c^2}\geq 1\)

BĐT cần chứng minh tương đương với \(\frac{\frac{1}{c^3}}{\frac{1}{a^2}+\frac{1}{b^2}}+\frac{\frac{1}{b^3}}{\frac{1}{a^2}+\frac{1}{c^2}}+\frac{\frac{1}{a^3}}{\frac{1}{b^2}+\frac{1}{c^2}}\geq \frac{\sqrt{3}}{2}\)

Đặt \((\frac{1}{a},\frac{1}{b},\frac{1}{c})=(x,y,z)\). Bài toán trở thành: 

Cho \(x,y,z>0|x^2+y^2+z^2\geq 1\). CMR \(P=\frac{x^3}{y^2+z^2}+\frac{y^3}{z^2+x^2}+\frac{z^3}{x^2+y^2}\geq \frac{\sqrt{3}}{2}\)

Lời giải:

 Áp dụng BĐT Cauchy -Schwarz:

\(P=\frac{x^4}{xy^2+xz^2}+\frac{y^4}{yz^2+yx^2}+\frac{z^4}{zx^2+zy^2}\geq \frac{(x^2+y^2+^2)^2}{x^2(y+z)+y^2(x+z)+z^2(x+y)}\) (1)

Không mất tính tổng quát, giả sử \(x\geq y\geq z\Rightarrow x^2\geq y^2\geq z^2\) 

Và \(y+z\leq z+x\leq x+y\). Khi đó, áp dụng BĐT Chebyshev: 

\(3[x^2(y+z)+y^2(x+z)+z^2(x+y)]\leq (x^2+y^2+z^2)(y+z+x+z+x+y)\)

\(\Leftrightarrow x^2(y+z)+y^2(x+z)+z^2(x+y)\leq \frac{2(x^2+y^2+z^2)(x+y+z)}{3}\)

Theo hệ quả của BĐT Am-Gm thì: \((x+y+z)^2\leq 3(x^2+y^2+z^2)\Rightarrow x+y+z\leq \sqrt{3(x^2+y^2+z^2)}\)

\(\Rightarrow x^2(y+z)+y^2(x+z)+z^2(x+y)\leq \frac{2(x^2+y^2+z^2)\sqrt{3(x^2+y^2+z^2)}}{3}\) (2)

Từ (1),(2) suy ra \(P\geq \frac{3(x^2+y^2+z^2)^2}{2(x^2+y^2+z^2)\sqrt{3(x^2+y^2+z^2)}}=\frac{\sqrt{3(x^2+y^2+z^2)}}{2}\geq \frac{\sqrt{3}}{2}\)

Ta có đpcm

Dáu bằng xảy ra khi \(x=y=z=\frac{1}{\sqrt{3}}\Leftrightarrow a=b=c=\sqrt{3}\)

5 tháng 5 2020

Đặt \(x=\frac{1}{a};y=\frac{1}{b};z=\frac{1}{c}\)

Khi đó giả thiết được viết lại là \(x^2+y^2+z^2\ge1\)và ta cần chứng minh \(\frac{x^3}{y^2+z^2}+\frac{y^3}{z^2+x^2}+\frac{z^3}{x^2+y^2}\ge\frac{\sqrt{3}}{2}\)(*)

Áp dụng BĐT Bunhiacopxki dạng phân thức, ta được:

\(VT_{\left(^∗\right)}=\frac{x^4}{x\left(y^2+z^2\right)}+\frac{y^4}{y\left(z^2+x^2\right)}+\frac{z^4}{z\left(x^2+y^2\right)}\)\(\ge\frac{\left(x^2+y^2+z^2\right)^2}{x\left(y^2+z^2\right)+y\left(z^2+x^2\right)+z\left(x^2+y^2\right)}\)

Đến đây ta đi chứng minh \(\frac{\left(x^2+y^2+z^2\right)^2}{x\left(y^2+z^2\right)+y\left(z^2+x^2\right)+z\left(x^2+y^2\right)}\ge\frac{\sqrt{3}}{2}\)

\(\Leftrightarrow2\left(x^2+y^2+z^2\right)^2\)\(\ge\sqrt{3}\left[x\left(y^2+z^2\right)+y\left(z^2+x^2\right)+z\left(x^2+y^2\right)\right]\)

Ta có: \(x\left(y^2+z^2\right)=\frac{1}{\sqrt{2}}\sqrt{2x^2\left(y^2+z^2\right)\left(y^2+z^2\right)}\)\(\le\frac{1}{\sqrt{2}}\sqrt{\left(\frac{2x^2+y^2+z^2+y^2+z^2}{3}\right)^3}\)

\(=\frac{2\sqrt{3}}{9}\left(x^2+y^2+z^2\right)\sqrt{x^2+y^2+z^2}\)

Tương tự ta có: \(y\left(z^2+x^2\right)\le\frac{2\sqrt{3}}{9}\left(x^2+y^2+z^2\right)\sqrt{x^2+y^2+z^2}\)

\(z\left(x^2+y^2\right)\le\frac{2\sqrt{3}}{9}\left(x^2+y^2+z^2\right)\sqrt{x^2+y^2+z^2}\)

Cộng theo vế của 3 BĐT trên, ta được: 

\(\text{∑}_{cyc}\left[x\left(y^2+z^2\right)\right]\le\frac{2\sqrt{3}}{3}\left(x^2+y^2+z^2\right)\sqrt{x^2+y^2+z^2}\)

\(\Leftrightarrow\sqrt{3}\text{∑}_{cyc}\left[x\left(y^2+z^2\right)\right]\le2\left(x^2+y^2+z^2\right)\sqrt{x^2+y^2+z^2}\)

Cuối cùng ta cần chứng minh được

\(2\left(x^2+y^2+z^2\right)\sqrt{x^2+y^2+z^2}\le2\left(x^2+y^2+z^2\right)^2\)

\(\Leftrightarrow x^2+y^2+z^2\ge1\)(đúng)

Đẳng thức xảy ra khi \(x=y=z=\frac{1}{\sqrt{3}}\Rightarrow a=b=c=\sqrt{3}\)

30 tháng 5 2020

Ta có: \(a^2+2b+3=\left(a^2+1\right)+2\left(b+1\right)\ge2\left(a+b+1\right)\)

Tương tự ta có: \(b^2+2c+3\ge2\left(b+c+1\right)\)\(c^2+2a+3\ge2\left(c+a+1\right)\)

Từ đó suy ra\(\frac{a}{a^2+2b+3}+\frac{b}{b^2+2c+3}+\frac{c}{c^2+2a+3}\)\(\le\frac{a}{2\left(a+b+1\right)}+\frac{b}{2\left(b+c+1\right)}+\frac{c}{2\left(c+a+1\right)}\)

\(=\frac{1}{2}\left(\frac{a}{a+b+1}+\frac{b}{b+c+1}+\frac{c}{c+a+1}\right)\)

Đặt \(K=\frac{a}{a+b+1}+\frac{b}{b+c+1}+\frac{c}{c+a+1}\), ta đi chứng minh \(K\le1\)

Thật vậy: \(3-K=\frac{b+1}{a+b+1}+\frac{c+1}{b+c+1}+\frac{a+1}{c+a+1}\)

\(=\frac{\left(b+1\right)^2}{\left(b+1\right)\left(a+b+1\right)}+\frac{\left(c+1\right)^2}{\left(c+1\right)\left(b+c+1\right)}+\frac{\left(a+1\right)^2}{\left(a+1\right)\left(c+a+1\right)}\)

\(\ge\frac{\left(a+b+c+3\right)^2}{\left(b+1\right)\left(a+b+1\right)+\left(c+1\right)\left(b+c+1\right)+\left(a+1\right)\left(c+a+1\right)}\)(*)

Ta có: \(\left(b+1\right)\left(a+b+1\right)+\left(c+1\right)\left(b+c+1\right)+\left(a+1\right)\left(c+a+1\right)\)\(=3\left(a+b+c\right)+ab+bc+ca+a^2+b^2+c^2+3\)

(Mình gõ bằng chương trình Universal Math Solver, không hiện ảnh thì vô thống kê hỏi đáp của mình, ngày 30/5/2020 vào lúc 8:25)

\(=\frac{1}{2}\left[\left(a+b+c\right)^2+6\left(a+b+c\right)+9\right]=\frac{1}{2}\left(a+b+c+3\right)^2\)(**)

Từ (*) và (**) suy ra \(3-K\ge\frac{\left(a+b+c+3\right)^2}{\frac{1}{2}\left(a+b+c+3\right)^2}=2\Rightarrow K\le1\)

Vậy ta có điều phải chứng minh

Đẳng thức xảy ra khi a = b = c = 1

30 tháng 5 2020

Áp dụng BĐT Cô-si,ta có :

\(a^2+1\ge2a\)

\(\Rightarrow\frac{a}{a^2+2b+3}\le\frac{a}{2a+2b+2}=\frac{1}{2}\left(\frac{a}{a+b+1}\right)\)

Tương tự : \(\frac{b}{b^2+2c+3}\le\frac{1}{2}\left(\frac{b}{b+c+1}\right);\frac{c}{c^2+2a+3}\le\frac{1}{2}\left(\frac{c}{c+a+1}\right)\)

\(\Rightarrow\frac{a}{a^2+2b+3}+\frac{b}{b^2+2c+3}+\frac{c}{c^2+2a+3}\le\frac{1}{2}\left(\frac{a}{a+b+1}+\frac{b}{b+c+1}+\frac{c}{c+a+1}\right)\)

Áp dụng BĐT Bu-nhi-a-cốp-ski,ta có :

\(\frac{a}{a+b+1}=\frac{a\left(a+b+c^2\right)}{\left(a+b+1\right)\left(a+b+c^2\right)}\le\frac{a^2+ab+ac^2}{\left(a^2+b^2+c^2\right)^2}=\frac{a^2+ab+ac^2}{9}\)

TT : ...

Cộng lại ta được :

\(\frac{a}{a+b+1}+\frac{b}{b+c+1}+\frac{c}{c+a+1}\le\frac{a^2+ab+ac^2}{9}+\frac{b^2+bc+ba^2}{9}+\frac{c^2+ca+cb^2}{9}\)

\(=\frac{a^2+b^2+c^2+ab+bc+ac+ac^2+ba^2+cb^2}{9}\le\frac{3+3+3}{9}=1\)

\(\Rightarrow\frac{a}{a^2+2b+3}+\frac{b}{b^2+2c+3}+\frac{c}{c^2+2a+3}\le\frac{1}{2}\)

Dấu "=" xảy ra khi a = b = c = 1

Ta có :

\(\(a^2+b^2+c^2=3\ge\frac{1}{3}\left(a+b+c\right)^2\Rightarrow a+b+c\le3\)\)

+) \(\(\frac{2a^2}{a+b^2}+\frac{2b^2}{b+c^2}+\frac{2c^2}{c+a^2}=\frac{4a^4}{2a^3+2a^2b^2}+\frac{4b^4}{2b^3+2b^2c^2}+\frac{4c^4}{2c^3+2c^2a^2}\)\)

\(\(\ge\frac{4\left(a^2+b^2+c^2\right)^2}{2a^3+2b^3+2c^3+2a^2b^2+2b^2c^2+2c^2a^2}\)\)

\(\(\ge\frac{4.3^2}{a^4+a^2+b^4+b^2+c^4+c^2+2a^2b^2+2b^2c^2+2c^2a^2}\)\)

\(\(=\frac{36}{\left(a^2+b^2+c^2\right)^2+a^2+b^2+c^2}=\frac{36}{9+3}=3\ge a+b+c\left(dpcm\right)\)\)

_Minh ngụy_

Dễ thấy 

\(3=a^2+b^2+c^2\ge\frac{1}{3}\left(a+b+c\right)^2\)

\(\Rightarrow a+b+c\le3\)

Do đó : 

\(\frac{2a^2}{a+b^2}+\frac{2b^2}{b+c^2}+\frac{2c^2}{c+a^2}=\frac{4a^4}{2a^3+2a^2b^2}+\frac{4b^4}{2b^3+2b^2c^2}+\frac{4c^4}{2c^3+2c^2a^2}\)

\(\ge\frac{\left(2a^2+2b^2+2c^2\right)^2}{2a^3+2b^3+2c^3+2a^2b^2+2b^2c^2+2c^2a^2}\)

\(\ge\frac{36}{a^4+a^2+b^4+b^2+c^4+c^2+2a^2b^2+2b^2c^2+2c^2a^2}\)

\(=\frac{36}{\left(a^2+b^2+c^2\right)^2+a^2+b^2+c^2}=3\ge a+b+c\left(dpcm\right)\)

4 tháng 9 2018

\(\frac{2a^2}{a+b^2}=2a-\frac{2ab^2}{a+b^2}\ge2a-\frac{2ab^2}{2b\sqrt{a}}=2a-b\sqrt{a}\ge2a-\frac{b+ba}{2}\) 

Tương tự rồi cộng từng vế ta có: 

\(\frac{2a^2}{a+b^2}+\frac{2b^2}{b+c^2}+\frac{2c^2}{c+a^2}\ge\frac{3}{2}\left(a+b+c\right)-\frac{ab+bc+ca}{2}\) 

Lại có: \(\left(a+b+c\right)^2\left(a^2+b^2+c^2\right)\ge3\left(ab+bc+ca\right)^2\Rightarrow a+b+c\ge ab+bc+ca\) 

\(\Rightarrow VT\ge\frac{3}{2}\left(a+b+c\right)-\frac{a+b+c}{2}\ge a+b+c\) 

Dấu "=' khi a=b=c=1

11 tháng 6 2020

Làm 2 cách nhá 

\(\frac{2a^2}{a+b^2}=\frac{2a^2}{\frac{a^2+1}{2}+b^2}=\frac{4a^2}{a^2+2b^2+1}=\frac{4a^4}{a^4+2a^2b^2+a^2}\)

Tương tự rồi theo Cauchy Schwarz ta có được:

\(LHS\ge\frac{\left(2a^2+2b^2+2c^2\right)^2}{a^4+b^4+c^4+2a^2b^2+2b^2c^2+2c^2a^2+3}=\frac{36}{\left(a^2+b^2+c^2\right)^2+3}=\frac{36}{12}=3\)

Đẳng thức xảy ra tại a=b=c=1

8 tháng 2 2017

\(BDT\LeftrightarrowΣ\frac{a^2}{a+b^2}\ge\frac{a+b+c}{2}\)

Áp dụng BDT C-S dạng Engel ta có:

\(Σ\frac{a^2}{a+b^2}=\text{ }Σ\frac{a^4}{a^3+a^2b^2}\ge\frac{\left(a^2+b^2+c^2\right)^2}{Σa^3+a^2b^2}\)

Vậy đi chứng minh \(\frac{\left(a^2+b^2+c^2\right)^2}{Σa^3+a^2b^2}\ge\frac{a+b+c}{2}\)

Hay \(2\left(a^2+b^2+c^2\right)^2\ge\left(a+b+c\right)Σ\left(a^3+b^2c^2\right)\)

\(\hept{\begin{cases}a+b+c=3u\\ab+ac+bc=3v^2\\abc=w^3\end{cases}}\)

Bởi vì điều kiện không phụ thuộc vào \(w^3\), ta thấy rằng bất đẳng thức cuối cùng là một bất đẳng thức tuyến tính của \(w^3\), đủ để chứng minh rằng bất đẳng thức cuối cùng đạt một giá trị cực đại là \(w^3\), xảy ra trong trường hợp hai biến bằng nhau hoặc có thể cho \(w^3\rightarrow0^+\)

Sau khi biến đổi đồng nhất ta cần chứng minh.

 

    \(\left(2\left(a^2+b^2+c^2\right)^2-\left(a+b+c\right)\left(a^3+b^3+c^3\right)\right)^2\left(a^2+b^2+c^2\right)\)

    \(\ge3\left(a+b+c\right)^2\left(a^2b^2+a^2c^2+b^2c^2\right)^2\)

    *)\(b=c=1\) Ta được

    \(\left(a-1\right)^2\left(a^8-2a^7+17a^6-8a^5+75a^4-10a^3+73a^2-4a+20\right)\ge0\) ( hiển nhiên đúng)

    *)\(w^3\rightarrow0^+\) để  \(c\rightarrow0^+\) và \(b=1\), ta đc:

    \(a^{10}-2a^9+10a^8-12a^7+26a^6-26a^5+26a^4-12a^3+10a^2-2a+1\ge0\)( cũng đúng)

    8 tháng 2 2017

    cách này phiêu quá lát mk làm lại

    25 tháng 8 2017

    Ta có BĐT \(a+b+c\le\sqrt{3\left(a^2+b^2+c^2\right)}=3\)

    Nên BĐT cần chứng minh là 

    \(\frac{a^2}{a+b^2}+\frac{b^2}{b+c^2}+\frac{c^2}{c+a^2}\ge\frac{3}{2}\)

    Đặt \(\hept{\begin{cases}a^2=x\\b^2=y\\c^2=z\end{cases}}\)\(\Rightarrow\hept{\begin{cases}x+y+z=3\\x,y,z>0\end{cases}}\)

    Áp dụng BĐT AM-GM and Cauchy-Schwarz ta có:

    \(Σ\frac{a^2}{a+b^2}=Σ\frac{x}{\sqrt{x}+y}=Σ\frac{x}{\sqrt{\frac{x\left(x+y+z\right)}{3}+y}}\)

    \(=Σ\frac{6x}{2\sqrt{3x\left(x+y+z\right)}+6y}\geΣ\frac{6x}{3x+x+y+z+6y}=Σ\frac{6x}{4x+7y+z}\)

    \(=Σ\frac{6x^2}{4x^2+7xy+xz}\ge\frac{6\left(x+y+z\right)^2}{Σ\left(4x^2+7xy+xz\right)}=\frac{3}{2}\)

    -Nguồn : Xem câu hỏi